LSAT and Law School Admissions Forum

Get expert LSAT preparation and law school admissions advice from PowerScore Test Preparation.

User avatar
 Dave Killoran
PowerScore Staff
  • PowerScore Staff
  • Posts: 5853
  • Joined: Mar 25, 2011
|
#82650
Complete Question Explanation
(The complete setup for this game can be found here: lsat/viewtopic.php?t=867)

The correct answer choice is (E).

This is an easy question to solve; simply apply the rules in the question stem. The question stem indicates that two rounds have been played, starting with an odd position round. If the lower-positioned team won every match, then after the first round the positions would have appeared as follows:

G4-Q20-d1.png

The next round would be even, and if every lower-positions team won, the results would appear as follows:

G4-Q20-d2.png

Accordingly, answer choice (E) is correct.
You do not have the required permissions to view the files attached to this post.
 ValVal
  • Posts: 8
  • Joined: Jul 05, 2017
|
#42656
Hi guys!

I cannot wrap my mind around Q20.

I think I know how to diagram, but it does not seem to accord the answer choices. So here is my train of thought:
We have 2 rounds of matches, beginning w/ odd one, and lower position teams always won.

Since it is odd position match, either 3-2 or 5-4 round can be a starting point:

1. If 3-2: we would have RSJML. Then second round could be either:
a. 2-1, then the teams will be situated as: SRJML; or
b. 4-3, then the teams will be situated as: RSMJL.

2. If we start w/ 5-4, then: RJSLM. The second round:
a. 2-1, then: JRSLM; or
b. 4-3, then: RJLSM.

It seems like a right approach to me. However, both (A) and (B) seem to fit as a right answer.
Would you help me to clarify this?
 Adam Tyson
PowerScore Staff
  • PowerScore Staff
  • Posts: 5153
  • Joined: Apr 14, 2011
|
#42672
I think you may be misunderstanding how the rounds work in this game, ValVal. In an odd-numbered round, teams 2 and 3 play each other AND teams 4 and 5 play each other. Two games happen, four teams play, and one team has a "bye" and sits out. In an odd round, team 1 gets the bye and sits out, while in an even round, 1 plays 2, 3 plays 4, and 5 sits out.

Here, we start with an order of RJSML, and we play an odd round (J plays S, M plays L, R sits out) and the two lower positioned teams in each pair (S and L) win and move up, swapping positions with their opponents. After that first odd round, the order is now RSJLM. Next we play an even round, with 1 playing 2 and 3 playing 4, with 2 and 4 winning and each moving up and swapping position with their opponents, and 5 sits out. The result after this second round is thus SRLJM. Answers A through D all must be true, while answer E is false - M is in 5th position, not 3rd.

The key is that in each round, two games are being played (picture 5 teams at two tennis courts). You have it as choosing one pair to play each round, but it's both pairs playing simultaneously.

Attack the game again with that new understanding, and see how it affects your outcomes. Good luck!

Get the most out of your LSAT Prep Plus subscription.

Analyze and track your performance with our Testing and Analytics Package.